Đến nội dung

Hình ảnh

Topic tổng hợp một số bất đẳng thức trong kì thi MO các nước

* * * * * 16 Bình chọn

  • Please log in to reply
Chủ đề này có 501 trả lời

#121
khanghaxuan

khanghaxuan

    Trung úy

  • Thành viên
  • 969 Bài viết

Đây là file tổng hợp khoảng 50 bài đầu trong topic :)

P/s : Mong các bạn tiếp tục tham gia sôi nổi :))

File gửi kèm


Bài viết đã được chỉnh sửa nội dung bởi khanghaxuan: 24-05-2015 - 14:17

Điều tôi muốn biết trước tiên không phải là bạn đã thất bại ra sao mà là bạn đã chấp nhận nó như thế nào .

- A.Lincoln -

#122
nhungvienkimcuong

nhungvienkimcuong

    Thiếu úy

  • Hiệp sỹ
  • 678 Bài viết

Đây là file tổng hợp khoảng 50 bài đầu trong topic :)

P/s : Mong các bạn tiếp tục tham gia sôi nổi :))

còn vài bài chưa có lời giải mà nhỉ


Đừng khóc vì chuyện đã kết thúc hãy cười vì chuyện đã xảy ra ~O) 
Thật kì lạ anh không thể nhớ đến tên mình mà chỉ nhớ đến tên em :wub:
Chúa tạo ra vũ trụ của con người còn em tạo ra vũ trụ của anh :ukliam2:


#123
khanghaxuan

khanghaxuan

    Trung úy

  • Thành viên
  • 969 Bài viết

Bài 50 : Cho $x,y,z$ là các số thực dương thỏa mãn : $(xy)^{2}+(yz)^{2}+(zx)^{2}=6xyz$ . CMR : 

$\sqrt{\frac{x}{x+yz}}+\sqrt{\frac{y}{y+zx}}+\sqrt{\frac{z}{z+xy}}\geq \sqrt{3}$

 

Bài 51 : Cho $x,y,z$ là các số thực dương . Chứng minh rằng : 

$\sum \frac{xy}{xy+x^{2}+y^{2}}\leq \sum \frac{x}{2x+z}$


Điều tôi muốn biết trước tiên không phải là bạn đã thất bại ra sao mà là bạn đã chấp nhận nó như thế nào .

- A.Lincoln -

#124
hoanglong2k

hoanglong2k

    Trung úy

  • Điều hành viên THCS
  • 965 Bài viết

Bài 50 : Cho $x,y,z$ là các số thực dương thỏa mãn : $(xy)^{2}+(yz)^{2}+(zx)^{2}=6xyz$ . CMR : 

$\sqrt{\frac{x}{x+yz}}+\sqrt{\frac{y}{y+zx}}+\sqrt{\frac{z}{z+xy}}\geq \sqrt{3}$

 

 Đặt $(a,b,c)=\left ( \frac{xy}{z},\frac{yz}{x},\frac{xz}{y} \right )$ thì từ giả thiết sẽ có $a+b+c=6$

 Ta cần chứng minh $\sum \sqrt{\frac{1}{1+a}}\geq \sqrt3$

 Áp dụng AM-GM thì $\sum \sqrt{\frac{1}{1+a}}\geq 3\sqrt[6]{\frac{1}{\prod (1+a)}}\geq 3\sqrt[6]{\frac{27}{(3+\sum a)^3}}=\sqrt3$

 Dấu "=" xảy ra khi $a=b=c=2<=> x=y=z=2$



#125
binhnhaukhong

binhnhaukhong

    Sĩ quan

  • Thành viên
  • 343 Bài viết

Bài 51:

 

KMTTQ,giả sử $x\geq y\geq z$,BĐT của ta tương đương:

 

$\sum \frac{(x-y)^2}{x^2+xy+y^2}+3\sum \frac{x}{2x+y}\geq 3$

Ta sử dụng BĐT C-S,có:

 

$\sum \frac{(x-y)^2}{x^2+xy+y^2}\geq \frac{4(x-z)^2}{\sum (x^2+xy+y^2)}$

$\sum \frac{x}{2x+y}\geq \frac{(x+y+z)^2}{\sum x(2x+y)}$

 

Vậy ta quy về chứng minh:

 

$4(x-y)^2+3(x+y+z)^2\geq 6(x^2+y^2+z^2)+3(xy+yz+xz)$

$\Leftrightarrow (x-z)^2+3(x-y)(y-z)\geq 0$

 

Đúng theo giả sử.

(Còn 1 cách nữa bằng khai triển nhưng dài quá )


Bài viết đã được chỉnh sửa nội dung bởi binhnhaukhong: 24-05-2015 - 18:26

Quy Ẩn Giang Hồ. 

So goodbye!

 

:off:  :off:  :off:  :off:  :off:  :off: 


#126
nhungvienkimcuong

nhungvienkimcuong

    Thiếu úy

  • Hiệp sỹ
  • 678 Bài viết

Bài 52:(IMO Shortlist 2009)

Cho $a,b,c>0$ mà $\frac{1}{a}+\frac{1}{b}+\frac{1}{c}\leq 3$.CMR

$\sqrt{\frac{a^2+b^2}{a+b}}+\sqrt{\frac{b^2+c^2}{b+c}}+\sqrt{\frac{c^2+a^2}{c+a}}+3\leq \sqrt{2}\left ( \sqrt{a+b}+\sqrt{b+c}+\sqrt{c+a} \right )$

 

Spoiler

Spoiler


Bài viết đã được chỉnh sửa nội dung bởi nhungvienkimcuong: 24-05-2015 - 19:09

Đừng khóc vì chuyện đã kết thúc hãy cười vì chuyện đã xảy ra ~O) 
Thật kì lạ anh không thể nhớ đến tên mình mà chỉ nhớ đến tên em :wub:
Chúa tạo ra vũ trụ của con người còn em tạo ra vũ trụ của anh :ukliam2:


#127
Pham Quoc Thang

Pham Quoc Thang

    Trung sĩ

  • Thành viên
  • 160 Bài viết

Bài 52:(IMO Shortlist 2009)

Cho $a,b,c>0$ mà $\frac{1}{a}+\frac{1}{b}+\frac{1}{c}\leq 3$.CMR

$\sqrt{\frac{a^2+b^2}{a+b}}+\sqrt{\frac{b^2+c^2}{b+c}}+\sqrt{\frac{c^2+a^2}{c+a}}+3\leq \sqrt{2}\left ( \sqrt{a+b}+\sqrt{b+c}+\sqrt{c+a} \right )$

 

Bài 53:

Cho $a,b,c$ là các số thực dương thỏa mãn $a^2+b^2+c^2=3$.CMR

$\frac{1}{a^3b}+\frac{1}{b^3c}+\frac{1}{c^3a}\geq 3$

Spoiler

Bài 53:
$a^2+b^2+c^2=3$ nên $abc \leq 1$
$\frac{1}{a^3b}+\frac{1}{b^3c}+\frac{1}{c^3a} \geq 3\sqrt[3]{\frac{1}{a^4b^4c^4}} \geq 3 $



#128
nhungvienkimcuong

nhungvienkimcuong

    Thiếu úy

  • Hiệp sỹ
  • 678 Bài viết

Bài 53:
$a^2+b^2+c^2=3$ nên $abc \leq 1$
$\frac{1}{a^3b}+\frac{1}{b^3c}+\frac{1}{c^3a} \geq 3\sqrt[3]{\frac{1}{a^4b^4c^4}} \geq 3 $

làm phiền bạn nhưng mình đăng đề nhầm

xin lỗi


Đừng khóc vì chuyện đã kết thúc hãy cười vì chuyện đã xảy ra ~O) 
Thật kì lạ anh không thể nhớ đến tên mình mà chỉ nhớ đến tên em :wub:
Chúa tạo ra vũ trụ của con người còn em tạo ra vũ trụ của anh :ukliam2:


#129
Hoang Tung 126

Hoang Tung 126

    Thiếu tá

  • Thành viên
  • 2061 Bài viết

Bài 52:(IMO Shortlist 2009)

Cho $a,b,c>0$ mà $\frac{1}{a}+\frac{1}{b}+\frac{1}{c}\leq 3$.CMR

$\sqrt{\frac{a^2+b^2}{a+b}}+\sqrt{\frac{b^2+c^2}{b+c}}+\sqrt{\frac{c^2+a^2}{c+a}}+3\leq \sqrt{2}\left ( \sqrt{a+b}+\sqrt{b+c}+\sqrt{c+a} \right )$

 

 

 Ta có : $\sqrt{2(a^2+b^2)}+\sqrt{4ab}\leq \sqrt{2(2(a^2+b^2)+4ab)}=\sqrt{4(a+b)^2}=2(a+b)= > \sqrt{2(a+b)}\geq \sqrt{\frac{a^2+b^2}{a+b}}+\sqrt{\frac{2ab}{a+b}}= > \sqrt{2(a+b)}-\sqrt{\frac{a^2+b^2}{a+b}}\geq \sqrt{\frac{2ab}{a+b}}$

 

  Tương tự  $\sqrt{2(b+c)}-\sqrt{\frac{b^2+c^2}{b+c}}\geq \sqrt{\frac{2bc}{b+c}}$

                    $\sqrt{2(c+a)}-\sqrt{\frac{c^2+a^2}{c+a}}\geq \sqrt{\frac{2ac}{a+c}}$

 

 Cộng theo vế các BDT 

 

$= > \sqrt{2}\sum \sqrt{a+b}-\sum \sqrt{\frac{a^2+b^2}{a+b}}\geq \sum \sqrt{\frac{2ab}{a+b}}=\sqrt{2}\sum \sqrt{\frac{ab}{a+b}}=\sqrt{2}\sum \frac{1}{\sqrt{\frac{1}{a}+\frac{1}{b}}}\geq \sqrt{2}.\frac{9}{\sum \sqrt{\frac{1}{a}+\frac{1}{b}}}\geq \sqrt{2}.\frac{9}{\sqrt{3(\sum (\frac{1}{a}+\frac{1}{b}))}}=\frac{9\sqrt{2}}{\sqrt{6\sum \frac{1}{a}}}\geq \frac{9\sqrt{2}}{\sqrt{6.3}}=3= > \sqrt{2}\sum \sqrt{a+b}-\sum \sqrt{\frac{a^2+b^2}{a+b}}\geq 3= > \sqrt{2}(\sum \sqrt{a+b})\geq \sum \sqrt{\frac{a^2+b^2}{a+b}}+3$

 

  Do đó ta có ĐPCM .Dấu = xảy ra khi $a=b=c=1$

 

 

 



#130
binhnhaukhong

binhnhaukhong

    Sĩ quan

  • Thành viên
  • 343 Bài viết

Cách khác cho bài 51:

 

Đổi biến $(\frac{x}{y},\frac{y}{z},\frac{z}{x})\rightarrow (a,b,c)$

 

Khi đó ta cần chứng minh:

$\sum \frac{a}{a^2+a+1}\leq \sum \frac{a}{2a+1}$

 

 

Đặt $p=a+b+c,q=ab+bc+ac$ thì ta có $p,q\geq 3$ và $q^2\geq 3p$

 

BĐT của tương đương:

 

$p^3+3p^2q+6p^2+pq^2-15pq-27p+4q^3-27q\geq 0$

 

Nhóm lại ta có BĐT tương đương:

$p(p^2-9)+3pq(p-3)+6p(p-3)+pq(q-3)+3q(q^2-9)+q(q^2-3p)\geq 0$

BĐT trên đúng.


Bài viết đã được chỉnh sửa nội dung bởi binhnhaukhong: 24-05-2015 - 19:20

Quy Ẩn Giang Hồ. 

So goodbye!

 

:off:  :off:  :off:  :off:  :off:  :off: 


#131
hoanglong2k

hoanglong2k

    Trung úy

  • Điều hành viên THCS
  • 965 Bài viết

Cách khác cho bài 51:

 

Đổi biến $(\frac{x}{y},\frac{y}{z},\frac{z}{x})\rightarrow (a,b,c)$

 

Khi đó ta cần chứng minh:

$\sum \frac{a}{a^2+a+1}\leq $$\sum \frac{a}{2a+1}$

 

 

Đặt $p=a+b+c,q=ab+bc+ac$ thì ta có $p,q\geq 3$ và $q^2\geq 3p$

 

BĐT của tương đương:

 

$p^3+3p^2q+6p^2+pq^2-15pq-27p+4q^3-27q\geq 0$

 

Nhóm lại ta có BĐT tương đương:

$p(p^2-9)+3pq(p-3)+6p(p-3)+pq(q-3)+3q(q^2-9)+q(q^2-3p)\geq 0$

BĐT trên đúng.

Hình như chỗ đó phải là $\sum \frac{1}{a+2}$ chứ anh :mellow:



#132
binhnhaukhong

binhnhaukhong

    Sĩ quan

  • Thành viên
  • 343 Bài viết

Hình như chỗ đó phải là $\sum \frac{1}{a+2}$ chứ anh :mellow:

Chỗ đó là bạn khanghaxuan gõ nhầm đề em ạ.

Đề nó là thế này:

 

$\sum \frac{xy}{x^2+xy+y^2}\leq\sum  \frac{x}{2x+y}$


Quy Ẩn Giang Hồ. 

So goodbye!

 

:off:  :off:  :off:  :off:  :off:  :off: 


#133
Bui Ba Anh

Bui Ba Anh

    Thiếu úy

  • Thành viên
  • 562 Bài viết
Các bạn đăng bài sau khi có lời giải của bài đó có thể chỉnh sửa ghi thêm nguồn của bài đó từ nước nào đc k ????
NgọaLong

#134
khanghaxuan

khanghaxuan

    Trung úy

  • Thành viên
  • 969 Bài viết

Lời giải khác bài 51 : 

Đổi biến $(\frac{1}{a};\frac{1}{b};\frac{1}{c})\rightarrow (x;y;z)$

Do đó ĐPCM viết lại : $\sum (\sqrt{\frac{2(a+b)}{ab}}-\sqrt{\frac{a^{2}+b^{2}}{ab(a+b)}})\geq 3$

$\Leftrightarrow \sum (\frac{a^{2}+b^{2}+4ab}{(\sqrt{\frac{2(a^{2}+b^{2})}{ab}}+\sqrt{\frac{a^{2}+b^{2}}{ab(a+b)}})})\geq 3$

$\Leftrightarrow \sum (\frac{2(a+b)^{2}-a^{2}-b^{2}}{(\sqrt{2}(a+b)+\sqrt{a^{2}+b^{2}})(\sqrt{ab(a+b)})})\geq 3$

$\Leftrightarrow \sum \frac{a^{2}+b^{2}+4ab}{(\sqrt{2ab(a+b)^{2}}+(\sqrt{ab(a+b)}))(\sqrt{a+b})}\geq 3$

Mặt khác , ta có : $\sqrt{2ab(a+b)^{2}}+\sqrt{ab(a^{2}+b^{2})}=\sqrt{2ab(a+b)^{2}}+\sqrt{\frac{a^{2}+b^{2}}{2}.2ab}\leq \sqrt{((a+b)^{2}+2ab)(\frac{a^{2}+b^{2}}{2}+2ab)}\leq \frac{a^{2}+b^{2}+4ab}{\sqrt{2}}$

Nên ta cần chứng minh : $\sum \frac{\sqrt{2}}{\sqrt{a+b}}\geq 3$

Thật vậy , $\sum \sqrt{\frac{2}{a+b}}\geq \sqrt{2}.\frac{9}{\sum \sqrt{a+b}}\geq \sqrt{2}.\frac{9}{\sqrt{3(2a+2b+2c)}}\geq 3$

Vậy ta có ĐPCM :)


Điều tôi muốn biết trước tiên không phải là bạn đã thất bại ra sao mà là bạn đã chấp nhận nó như thế nào .

- A.Lincoln -

#135
khanghaxuan

khanghaxuan

    Trung úy

  • Thành viên
  • 969 Bài viết

Bài 53 : ( Mediterranean MO 2002 ) Cho $a,b,c$ không âm thỏa mãn $a^{2}+b^{2}+c^{2}=1$ . CMR : 

$\frac{a}{b^{2}+1}+\frac{b}{c^{2}+1}+\frac{c}{a^{2}+1}\geq \frac{3}{4}(a\sqrt{a}+b\sqrt{b}+c\sqrt{c})^{2}$

 

Bài 54 : ( IMO shortlist 2006 ) Cho $a,b,c$ là ba cạnh của tam giác . CMR : 

$\sum \frac{\sqrt{b+c-a}}{\sqrt{b}+\sqrt{c}-\sqrt{a}}\leq 3$


Điều tôi muốn biết trước tiên không phải là bạn đã thất bại ra sao mà là bạn đã chấp nhận nó như thế nào .

- A.Lincoln -

#136
Hoang Tung 126

Hoang Tung 126

    Thiếu tá

  • Thành viên
  • 2061 Bài viết

Bài 53 : ( Mediterranean MO 2002 ) Cho $a,b,c$ không âm thỏa mãn $a^{2}+b^{2}+c^{2}=1$ . CMR : 

$\frac{a}{b^{2}+1}+\frac{b}{c^{2}+1}+\frac{c}{a^{2}+1}\geq \frac{3}{4}(a\sqrt{a}+b\sqrt{b}+c\sqrt{c})^{2}$

 

Bài 53: Theo Bunhiacopxki ta có :

 

 $\frac{a}{b^2+1}+\frac{b}{c^2+1}+\frac{c}{a^2+1}=\frac{a^3}{a^2b^2+a^2}+\frac{b^3}{b^2c^2+b^2}+\frac{c^3}{c^2a^2+c^2}\geq \frac{(a\sqrt{a}+b\sqrt{b}+c\sqrt{c})^2}{(a^2+b^2+c^2)+(a^2b^2+b^2c^2+c^2a^2)}\geq \frac{(a\sqrt{a}+b\sqrt{b}+c\sqrt{c})^2}{a^2+b^2+c^2+\frac{(a^2+b^2+c^2)^2}{3}}=\frac{(a\sqrt{a}+b\sqrt{b}+c\sqrt{c})^2}{1+\frac{1}{3}}=\frac{3}{4}(a\sqrt{a}+b\sqrt{b}+c\sqrt{c})^2$

 

 Dấu = xảy ra khi $a=b=c=\frac{1}{\sqrt{3}}$



#137
Hoang Tung 126

Hoang Tung 126

    Thiếu tá

  • Thành viên
  • 2061 Bài viết

 

Bài 54 : ( IMO shortlist 2006 ) Cho $a,b,c$ là ba cạnh của tam giác . CMR : 

$\sum \frac{\sqrt{b+c-a}}{\sqrt{b}+\sqrt{c}-\sqrt{a}}\leq 3$

  Đặt $\sqrt{a}=x,\sqrt{b}=y,\sqrt{c}=z$

 

Từ đó $\sum \frac{\sqrt{b+c-a}}{\sqrt{b}+\sqrt{c}-\sqrt{a}}=\sum \frac{\sqrt{y^2+z^2-x^2}}{y+z-x}\leq \sqrt{3(\sum \frac{y^2+z^2-x^2}{(y+z-x)^2})}\leq 3< = > \sum \frac{y^2+z^2-x^2}{(y+z-x)^2}\leq 3< = > \sum (\frac{y^2+z^2-x^2}{(y+z-x)^2}-1)\leq 0< = > \sum \frac{(y^2+z^2-x^2)-(y+z-x)^2}{(y+z-x)^2}\leq 0< = > \sum \frac{-2x^2-2yz+2xz+2xy}{(y+z-x)^2}\leq 0< = > \sum \frac{(x-y)(x-z)}{(y+z-x)^2}\geq 0$

$< = > \frac{(x-y)(x-z)}{(y+z-x)^2}+\frac{(y-z)(y-x)}{(x+z-y)^2}+\frac{(z-x)(z-y)}{(x+y-z)^2}\geq 0$  (1)

 

   Không mất tổng quát giả sử $x\geq y\geq z> 0= > \frac{(z-x)(z-y)}{(x+y-z)^2}\geq 0$

 

Ta cần CM :$\frac{(x-y)(x-z)}{(y+z-x)^2}+\frac{(y-z)(y-x)}{(x+z-y)^2}\geq 0< = > (x-y)(\frac{x-z}{(y+z-x)^2}-\frac{y-z}{(x+z-y)^2})\geq 0< = > \frac{x-z}{(y+z-x)^2}-\frac{y-z}{(x+z-y)^2}\geq 0$  

(Do $x-y\geq 0$)

 

 BDT trên đúng do $x\geq y= > \frac{x-z}{(y+z-x)^2}\geq \frac{y-z}{(y+z-x)^2}= > \frac{x-z}{(y+z-x)^2}-\frac{y-z}{(x+z-y)^2}\geq (y-z)(\frac{1}{(y+z-x)^2}-\frac{1}{(x+z-y)^2}=\frac{(y-z)}{(y+z-x)^2(x+z-y)^2}((x+z-y)^2-(y+z-x)^2)=\frac{4z(y-z)(x-y)}{(y+z-x)^2(x+z-y)^2}\geq 0$

 (Đúng do $x\geq y\geq z= > (y-z)(x-y)\geq 0$)

 

  Do đó ta có ĐPCM .Dấu = xảy ra khi $x=y=z< = > a=b=c$


Bài viết đã được chỉnh sửa nội dung bởi Hoang Tung 126: 25-05-2015 - 12:15


#138
nhungvienkimcuong

nhungvienkimcuong

    Thiếu úy

  • Hiệp sỹ
  • 678 Bài viết

Bài 55:(Ailen 2009)

Cho $a,b,c$ là các số thực thỏa $\left\{\begin{matrix} a+b+c=0\\a^2+b^2+c^2=1 \end{matrix}\right.$

Chứng minh rằng

$a^2b^2c^2\leq \frac{1}{54}$

 

Bài 56:(Ấn Độ 2009)

Cho $a,b,c>0$ và thỏa mãn $a^3+b^3=c^3$.

Chứng minh rằng

$a^2+b^2-c^2>6(a-c)(b-c)$


Đừng khóc vì chuyện đã kết thúc hãy cười vì chuyện đã xảy ra ~O) 
Thật kì lạ anh không thể nhớ đến tên mình mà chỉ nhớ đến tên em :wub:
Chúa tạo ra vũ trụ của con người còn em tạo ra vũ trụ của anh :ukliam2:


#139
hoanglong2k

hoanglong2k

    Trung úy

  • Điều hành viên THCS
  • 965 Bài viết

Bài 55:(Ailen 2009)

Cho $a,b,c$ là các số thực thỏa $\left\{\begin{matrix} a+b+c=0\\a^2+b^2+c^2=1 \end{matrix}\right.$

Chứng minh rằng

$a^2b^2c^2\leq \frac{1}{54}$

 

Từ giả thiết $=> b+c=-a=> 1-a^2=b^2+c^2\geq \frac{1}{2}.(b+c)^2=\frac{a^2}{2}=>a^2\leq \frac{2}{3}$

Lại có : $(b+c)^2=b^2+c^2+2bc<=>a^2=1-a^2+2bc=>bc=a^2-\frac{1}{2}=>b^2c^2=\left ( a^2-\frac{1}{2} \right )^2$

Ta cần chứng minh $a^2\left ( a^2-\frac{1}{2}\right )^2\leq \frac{1}{54}$

                               $<=>\left ( a^2-\frac{1} 6\right )^2\left ( a^2-\frac{2}{3} \right )\leq 0$    đúng với $\forall a^2\in \left [ 0;\frac{2}{3} \right ]$ 


Bài viết đã được chỉnh sửa nội dung bởi hoanglong2k: 26-05-2015 - 10:18


#140
Pham Quoc Thang

Pham Quoc Thang

    Trung sĩ

  • Thành viên
  • 160 Bài viết

Bài 56:Do $a,b,c>0$ nên $c>a,c>b;(\frac{a}{c})^3+(\frac{b}{c})^3=1$
Đặt:$x=\frac{a}{c};y=\frac{b}{c} (0<x,y<1);x^3+y^3=1$
Dễ dàng chứng minh được:$1<x+y \leq \sqrt[3]{4} $
Đặt:$z=x+y \Rightarrow xy=\frac{z^3-1}{3z};x^2+y^2=\frac{z^3+2}{3z}$
Cần chứng minh:$a^2+b^2-c^2>6(a-c)(b-c) \Leftrightarrow x^2+y^2-1>6(x-1)(y-1) \Leftrightarrow \frac{z+2}{z-1} >6$
Xét hàm $f(z)=\frac{z+2}{z-1};z \in (1;\sqrt[3]{4}]$
Ta dễ dàng chứng minh $f(z)$ nghịch biến $\Rightarrow f(z) \geq f(\sqrt[3]{4}) >6 $






0 người đang xem chủ đề

0 thành viên, 0 khách, 0 thành viên ẩn danh